Abstract

Let be the field of all complex numbers, the space of all matrices over , and the subspace of consisting of all symmetric matrices. The map satisfies that is -potent in implying that is -potent in , where then there exist an invertible matrix and with such that for every . Moreover, the inductive method used in this paper can be used to characterise similar maps from to .

1. Introduction

Let be the field of all complex numbers, the space of all matrices over , the subspace of consisting of all triangular matrices, and the subspace of consisting of all symmetric matrices. For fixed integer , is called a -potent matrix if ; especially, is an idempotent matrix when . The map satisfies that is a -potent matrix in implying that is a -potent matrix in , where , is a kind of the so-called weak preservers. While replacing “implying that” with “if and only if,” is called strong preserver. Obviously, a strong preserver must be a weak preserver, while a weak preserver may not be a strong preserver.

The preserver problem in this paper is from LPPs but without linear assumption (more details about LPP in [13]). You and Wang characterized the strong -potence preservers from to in [4]; then Song and Cao extended the result to weak preservers from to in [5]. In [6], Wang and You characterized the strong -potence preservers from to . In this paper, the authors characterized the weak -potence preservers from to and proved the following theorem.

Theorem 1. Suppose satisfy that is a -potent matrix in implying that is a -potent matrix in , where . Then there exist invertible and with such that for every .

Furthermore, we can derive the following corollary from Theorem 1.

Corollary 2. Suppose satisfy that is a -potent matrix in implying that is a -potent matrix in , where . Then there exist invertible and with such that for every , where for some nonzero .

In fact, the proof of Theorem 1 through some adjustments is suitable for the weak -potence preserver from to , and more details can be seen in remarks.

2. Notations and Lemmas

denotes the set of all -potent matrices in , while . denotes the set of all complex number satisfying , . denotes matrices in with in and elsewhere, and denotes the unit matrix in . denotes the set of integer satisfy . denotes the general linear group consisting of all invertible matrices in . denotes an arbitrary diagonal matrix in . For , , and are orthogonal if . denotes the space of all matrices over . denotes the set of all maps satisfying that is a -potent matrix in implying that is a -potent matrix in , where .

For an arbitrary matrix , we denote by the term in position of , by the matrix with the term in its position equal to , where and . Moreover, we denote by the matrix with the term in its position equal to and terms elsewhere equal to . We especially simplify it with when , and for every . Naturally, for every .

Without fixing , also denotes a matrix in with in its position, where , , and , .

At first, we need the following Lemmas 3, 4, 5, and 7, which are about -potent matrices and orthogonal matrices.

Lemma 3 (see [2]). Suppose , , and  for every ; then and are orthogonal.

Lemma 4 ([7, Lemma 1]). Suppose , , , are mutually orthogonal nonzero -potent matrices; then there exists such that with for every .

Lemma 5. Suppose , , , , with , , for arbitrary nonzero with and , . Then , , and there exist , with such that and .

Proof. By the assumption of and , is idempotent. Denote this matrix by , and then we can get the following equation:
Since the matrices on both sides of satisfy the following equation: then the following matrix is -potent by the assumption of lemma:
We denote by the following matrix: then the following equation is obvious: Unfolding it, we get ; that is, , where is the coefficient matrix of for every .
Let , then we calculate it and get the following equations:
It is easy to get and the following equation: Note that the highest degree of in is ; then the highest degree of in is less or equal to for every with , and the highest degree of in is , where is the coefficient matrix of in and is the coefficient of in .
By the assumption of , we have and . Then the following equations are true: and , where the highest degree of is and is the coefficient matrix of .
Now, we calculate the upper left part of .
When , , of which the upper left part is . Then in the upper left part of , the highest degree of is , and the coefficient matrix is .
When , if appears in the left (or right) end of an additive item of , then the upper left part of this item is . So, the upper left part of is equal to the upper left part of ; that is, the upper left part is , and the highest degree of is with as the coefficient matrix of .
By the assumption of , we have .
By , we have , , and if and only if . When , we can get by , and by , where and satisfy ; that is, by , which is equivalent to . When , .

Remark 6. Replacing with in Lemma 5, we have implies or , and implies or . These cases will not appear in the proof of Theorem 1, but are necessary for the weak preservers from to .

Lemma 7. Suppose  for arbitrary , with , where is a map satisfying for every . Then there exists nonzero such that for every .

Proof. Since the trace of is equal to , then , or , especially, when equal to , with . Denote by , and by ; then we have or .(1)If , then , that is, ; (2)if , then . When , ; when , . But implies , or . It is a contradiction! So it is impossible that .
Hence, there exists nonzero such that for every .

We can prove the following Lemmas 8 and 9 similar as Lemmas 4 and  5 in [4].

Lemma 8 (see [4], Lemma 4). Suppose , and are orthogonal -potent matrices; then and are orthogonal.

Lemma 9 (see [4], Lemma 5). Suppose ; then are homogeneous; that is, for every and every .

Corollary 10. Suppose , , , and for every , , . Then and are orthogonal.

Proof. By the assumption and Lemma 9, we have , , . By Lemma 3, and are orthogonal.

Corollary 11. Suppose and for arbitrary diagonal matrix . Then for every , with , , where is only decided by and .

Proof. Let , , and ; then , and satisfy the assumption of Corollary 10, and and are orthogonal; that is, for some , , , and .
Since for arbitrary nonzero with , after applying , we have . By Lemma 5, , .
Let , where for every ; then is the function of , , and and denote by the value of on , , , , and .
Fix , , and and add a free variable to for some ; then becomes into a map of . Since for arbitrary and with , then by and , we can derive that . By Lemma 7, for fixed , , and ; that is, for arbitrary . Similarly, we can prove for arbitrary .
In fact, we have proved that and for arbitrary , and arbitrary ; then follows.
Since for fixed , , and with , , and arbitrary , , then implies . By Lemma 7, we can get for arbitrary and with ; that is, for arbitrary .
Until now, we have proved that for arbitrary ; that is, is only decided by and .

Remark 12. The proof of Corollary 11 presents the basic procedure of proof of Theorem 1. In order to decide the image of matrix , we use Corollary 10 and the images of and , which usually are diagonal matrices or some matrices with images already decided.
If is a weak preserver from to , then Corollary 11 is also true. Let , , and ; then we can prove similarly as proving , and . Since for arbitrary nonzero , then the following matrix is -potent: Remark 6 tells us that ,  , or ; that is, , or , . Similarly, we can prove , , or . Since is arbitrary, we set for convenience.
If ; then implies ; that is, , which is a contradiction. Hence, we proved that it is impossible or .
If and , then implies ; that is, , which is a contradiction. Hence, we proved that and , or and .

3. Proof of Theorem 1

Suppose , then we can derive Theorem 1 from Propositions 13, 14, and 16.

Proposition 13. Suppose , with ; then if and only if .

Proof. Suppose and for some , with . At first, we prove that for arbitrary . Since the equation is already true when , then we assume in the following proof.
Let , , and ; then it is easy to verify , , and satisfying the assumption of Corollary 10. So and are orthogonal. Moreover, we can derive from and . Let , then and are orthogonal -potent matrices. While implies ; then . There are two cases on .(1)If , then ; that is, ; (2)if , we can derive that from . Note that , so it is true that ; that is, . Finally, we can derive from and . At the same time, .
Anyway, for arbitrary .
Since for every nonzero with , then , and by . While the equation is equivalent to . Note that is the constant term of the equation; then by the infinite property of , and follows. Then we can derive which is a contradiction to the assumption.

Proposition 14. Suppose for every ; then for arbitrary .

Proof. The proof will be completed by induction on the following equation for arbitrary with for every : where .
When , (10) is equivalent to for arbitrary .
At first, by the assumption, it is already true that for every .
Suppose for every with ; then by the homogeneity of , we just need to prove the following equation for with : There are two cases on . (1) If , then there exists such that , and the following statements are true:
Note that and by the assumption; then the following statements are true:
Since , then , and follows.(2) If , then we have the following statements:
Since ; then by case 1, and follows. While , hence we get .
Anyway, we prove ; then by the induction, (10) is true for .
Suppose (10) is true for , then we prove the case on .
Let , , ; then we have and the following equation: We will prove the following equation which is equivalent to (10) on :
For arbitrary nonzero with , the following matrix is idempotent: where .
Note that and satisfy the following equation:
After applying on the above matrices, we have by the inductive assumption. Then because of the assumption of ; that is, (10) holds for .
Finally, we prove that for every by the induction.

Remark 15. If is a weak -potence preserver from to ; then Propositions 13 and 14 (replacing with for arbitrary in the proof of Proposition 14) hold since Corollary 10 is true under this assumption.

Proposition 16. Suppose for every , then there exist and such that for every .

Proof. The proof will be completed in the following 4 steps.
Step 1. , where for every :
Since is nonzero -potent, then we can derive from Lemma 4 that there exists such that for every , where . It is obvious that the following map and for every . Without loss of generality, we can assume .
Step 2. , for arbitrary diagonal matrix .
The proof of this step can be seen in Step 3, Section  3 in [5].
Step 3. for every .
Let , , and , we can derive the following equation from Step 2 and Corollary 10: where , , , , , with .
Note that for , with . In fact, and are all the eigenvalues of this matrix. Applying on the matrix , we have .
Since is fixed, then is the finite set which contains all of eigenvalues of , and there exists such that the trace of is for infinite choices of ; that is, there exist , with such that the traces of and are all equal to ; then we have the following equation: which is equivalent to where , for , .
Naturally, there are infinite choices of for fixed such that the above equation is true. If is equal to some , where , and are fixed, then we can derive from the following equation: that there are infinite choices of for constant if and only if . While and imply , which is a contradiction to , hence varies with .
Since and are all fixed numbers for fixed , then implies that there are at least two different values of for fixed and infinite choices of ; it is a contradiction. So and follows. Hence for every .
Step 4. for every .
After the discussion in Steps 1, 2, and 3, we already have the following equation: where , for every . Since the map , then we can assume without loss of generality.
The proof in this step will be completed by induction on the following equation for arbitrary with for every : where with .
When , (25) is equivalent to for arbitrary diagonal matrix and , with , since is homogeneous. The proof will be completed in the following and .(1) for every .
We already derive from Corollary 11 that for every , where is only decided by .
Suppose the map satisfies the following equation for every , then , and for arbitrary diagonal matrix and every , and .
Without loss of generality, we can assume for every and arbitrary .(2) Suppose for every , with ; then for every , with .
At first, we have to prove that for arbitrary nonzero and .
By the assumption, we already have the following equations:
Let , , and . Then the following statements are true where and are -potent.
Let , , and , then , , and satisfy the assumption of Corollary 10. Hence we get and are orthogonal; that is,
Similarly, we can derive the following equation from Corollary 10:
Comparing the above two equations, we have , , , and , that is, .
We will prove . For arbitrary nonzero with , let , and ; then implies ; that is, the following matrix is -potent since by the assumption by Lemma 5, . Hence we prove .
Now we prove .
By Corollary 11, we already have .
For arbitrary nonzero with , is idempotent.
After applying on the above matrices, we have .
Then by Lemma 5.
By the induction, we prove for every , with .(3) Suppose (25) is true for every with ; then we prove it holds on .
For arbitrary with for every , let , , , , , and satisfy the following equations: Then is idempotent for arbitrary nonzero with . Applying on it, we have . Let ; then by for every , we have .
Note that and by the assumption; then and are orthogonal by Corollary 10; that is, for some .
On the other hand, implies by . By Lemma 5, we can derive the following equations: that is, .
Let and satisfy the following equations: then we can prove
Comparing the above three sets of equations, we can get , which is equivalent to (25) on .
By the induction, we prove that for arbitrary .

Remark 17. If is a weak -potence preserver from to , then the proof in Steps 1, 2, and 3 of Proposition 16 holds, and we prove or in Step 4. We omit the detailed proof since the case on is totally the same after changing relevant notations.